consecutive integers

This topic has expert replies
Senior | Next Rank: 100 Posts
Posts: 84
Joined: Sat Oct 03, 2009 1:13 pm
Thanked: 11 times

consecutive integers

by djkvakin » Tue Mar 09, 2010 9:52 pm
If m, n, p, and q are distinct integers, are they consecutive?
(1) q-m=3

(2) m<n<q and m<p<q

OA is C. [spoiler]My answer is E because we don't have any limitation on the integers: even if we consider 1 and 2 together, all we know is that q is the largest, and m is the smallest. consider the list m n p q: 1 3 3 4. where am i wrong with this?[/spoiler]

User avatar
Community Manager
Posts: 1537
Joined: Mon Aug 10, 2009 6:10 pm
Thanked: 653 times
Followed by:252 members

by papgust » Tue Mar 09, 2010 10:02 pm
I guess you didn't read the question properly. The question says "m, n, p, and q are distinct integers".

Master | Next Rank: 500 Posts
Posts: 377
Joined: Wed Sep 03, 2008 9:30 am
Thanked: 15 times
Followed by:2 members

by schumi_gmat » Tue Mar 09, 2010 10:05 pm
djkvakin wrote:If m, n, p, and q are distinct integers, are they consecutive?
(1) q-m=3

(2) m<n<q and m<p<q

OA is C. [spoiler]My answer is E because we don't have any limitation on the integers: even if we consider 1 and 2 together, all we know is that q is the largest, and m is the smallest. consider the list m n p q: 1 3 3 4. where am i wrong with this?[/spoiler]
The answer is C.

It is mentioned that the numbers are distinct. so they cannot be same. hence the condition 1 3 3 4 is invalid.

B tells us the position of the numbers and A gives the range.

Hence C

User avatar
Junior | Next Rank: 30 Posts
Posts: 28
Joined: Fri Feb 26, 2010 7:57 pm
Thanked: 1 times

by scoowhoop » Wed Mar 24, 2010 12:00 pm
Given (1) and (2) couldn't the sequence still be either, mpnq or mnpq? I dont' see how C can be correct.

User avatar
GMAT Instructor
Posts: 3225
Joined: Tue Jan 08, 2008 2:40 pm
Location: Toronto
Thanked: 1710 times
Followed by:614 members
GMAT Score:800

by Stuart@KaplanGMAT » Wed Mar 24, 2010 12:37 pm
scoowhoop wrote:Given (1) and (2) couldn't the sequence still be either, mpnq or mnpq? I dont' see how C can be correct.
The problem is more with the wording of the question, which can be interpreted different ways.

If you interpret it as "consecutive in that order", then the answer is E.

However, a more general interpretation of "consecutive" (i.e. we just need the 4 integers to be consecutive, in no particular order) would lead to C.

For example, how would you answer the question:

"Are 2,4,5 and 3 consecutive integers?"

Under the first interpretation they're not, but under the second interpretation they are.
Image

Stuart Kovinsky | Kaplan GMAT Faculty | Toronto

Kaplan Exclusive: The Official Test Day Experience | Ready to Take a Free Practice Test? | Kaplan/Beat the GMAT Member Discount
BTG100 for $100 off a full course